Difference between revisions of "2003 AMC 12A Problems/Problem 24"

m (Solution)
Line 19: Line 19:
  
 
Note that the maximum occurs when <math>a=b</math>.
 
Note that the maximum occurs when <math>a=b</math>.
 +
 +
 +
==Solution 2 (More Algebraic)==
 +
Similar to the previous solution, we use our logarithmic rules and start off the same way.
 +
<cmath>\log_{a}a-\log_{a}b+\log_{b}b-\log_{b}a = 2-(\log_{a}b+\log_{b}a)</cmath>
 +
 +
However, now, we use a different logarithmic rule stating that <math>\log_{a}b</math> is simply equal to <math>\frac {\log_{10}b}{\log_{10}a}</math>. With this, we can rewrite our previous equation to give us <cmath>2-(\log_{a}b+\log_{b}a) = 2 - (\frac {\log_{10}b}{\log_{10}a} + \frac {\log_{10}a}{\log_{10}b})</cmath>.
 +
 +
We can now cross multiply to get that <cmath>2 - (\frac {\log_{10}b}{\log_{10}a} + \frac {\log_{10}a}{\log_{10}b}) = 2 - (\frac {2 * log_{10}b * \log_{10}a}{log_{10}b * \log_{10}a})</cmath> Finally, we cancel to get <math>2-2=0 \Rightarrow \boxed{\textbf{(B) 0}}.</math>
 +
 +
Solution by: armang32324
  
 
==Video Solution==
 
==Video Solution==

Revision as of 20:35, 6 April 2023

Problem

If $a\geq b > 1,$ what is the largest possible value of $\log_{a}(a/b) + \log_{b}(b/a)?$

$\mathrm{(A)}\ -2      \qquad \mathrm{(B)}\ 0     \qquad \mathrm{(C)}\ 2      \qquad \mathrm{(D)}\ 3      \qquad \mathrm{(E)}\ 4$

Solution

Using logarithmic rules, we see that

\[\log_{a}a-\log_{a}b+\log_{b}b-\log_{b}a = 2-(\log_{a}b+\log_{b}a)\] \[=2-(\log_{a}b+\frac {1}{\log_{a}b})\]

Since $a$ and $b$ are both greater than $1$, using AM-GM gives that the term in parentheses must be at least $2$, so the largest possible values is $2-2=0 \Rightarrow \boxed{\textbf{B}}.$

Note that the maximum occurs when $a=b$.


Solution 2 (More Algebraic)

Similar to the previous solution, we use our logarithmic rules and start off the same way. \[\log_{a}a-\log_{a}b+\log_{b}b-\log_{b}a = 2-(\log_{a}b+\log_{b}a)\]

However, now, we use a different logarithmic rule stating that $\log_{a}b$ is simply equal to $\frac {\log_{10}b}{\log_{10}a}$. With this, we can rewrite our previous equation to give us \[2-(\log_{a}b+\log_{b}a) = 2 - (\frac {\log_{10}b}{\log_{10}a} + \frac {\log_{10}a}{\log_{10}b})\].

We can now cross multiply to get that \[2 - (\frac {\log_{10}b}{\log_{10}a} + \frac {\log_{10}a}{\log_{10}b}) = 2 - (\frac {2 * log_{10}b * \log_{10}a}{log_{10}b * \log_{10}a})\] Finally, we cancel to get $2-2=0 \Rightarrow \boxed{\textbf{(B) 0}}.$

Solution by: armang32324

Video Solution

The Link: https://www.youtube.com/watch?v=InF2phZZi2A&t=1s

-MistyMathMusic

See Also

2003 AMC 12A (ProblemsAnswer KeyResources)
Preceded by
Problem 23
Followed by
Problem 25
1 2 3 4 5 6 7 8 9 10 11 12 13 14 15 16 17 18 19 20 21 22 23 24 25
All AMC 12 Problems and Solutions

The problems on this page are copyrighted by the Mathematical Association of America's American Mathematics Competitions. AMC logo.png